Đến nội dung

Chris yang

Chris yang

Đăng ký: 26-03-2014
Offline Đăng nhập: Riêng tư
****-

#665306 $\int\frac{dx}{2^{x}+1}$

Gửi bởi Chris yang trong 20-12-2016 - 22:57

Tìm A = $\int\frac{dx}{2^{x}+1}$

Đặt $t=2^x+1$ thì $dt=\ln 2 (t-1)dx$

$\Rightarrow A=\frac{1}{\ln 2}\int \frac{dt}{t(t-1)}=\frac{1}{\ln 2}\int \left ( \frac{1}{t-1}-\frac{1}{t} \right )dt=\frac{1}{\ln 2}(\ln|t-1|-\ln|t|)+c$




#658153 Tìm tất cả các số nguyên dương $(q, r, p),$ với $p$ là số...

Gửi bởi Chris yang trong 16-10-2016 - 22:02

Tìm tất cả các số nguyên dương $(q, r, p),$ với $p$ là số nguyên tố thỏa mãn $2^{q}+r^{2}=2p.$

Dễ thấy $p$ lẻ

Từ PT suy ra $r$ chẵn $\Rightarrow 4|r^2$. Thấy $2^{q-1}=p-\frac{r^2}{2}$ lẻ do $p$ lẻ nên $q-1=0\Rightarrow q=1$

Khi đó $r^2=2(p-1)\Rightarrow p-1=2^{2k+1}x^2$ với $k,x\in\mathbb{N}$ hay $p$ có dạng $2^{2k+1}x^2+1$ ( Theo nguyên lí Dirichlet về sự tồn tại vô số số nguyên tố dạng $ak+b$ thì $p$ có vô số giá trị thỏa mãn $3,19,73,163,883....$)

 Vậy $(p,q,r)=(2^{2k+1}x^2+1,1,2^{k+1}x)$ với $k,x\in\mathbb{N}$




#658013 Tìm các số a,b sao cho $a^b=b^a$

Gửi bởi Chris yang trong 16-10-2016 - 00:08

Tìm các số số tự nhiên $a,b$ sao cho $a^b=b^a$

Với $a=b\in\mathbb{N}$ thì bài toán luôn đúng. Xét $a\neq b$. Giả sử $a>b$.

Đặt $a=p_1^{m_1}p_2^{m_2}....p_k^{m_k}$ với $p_i\in\mathbb{P}$ và $m_i\in\mathbb{N}$. Từ đó kéo theo $b=p_1^{n_1}p_2^{n_2}....p_k^{n_k}$

Vì $a^b=b^a$ nên $\frac{a}{b}=\frac{m_i}{n_i}=\prod_{1}^{k}p_i^{m_i-n_i}>1$

Giả sử $p$ là ước nguyên tố lớn nhất của $a$ và $b$, hiển nhiên $\frac{m}{n}\geq p^{m-n}\Rightarrow m\geq np^{m-n}$

$\Rightarrow m-n\geq n(p^{m-n}-1)\geq p^{m-n}-1$ $(\star)$

Ta sẽ CM với $x\geq 1\in\mathbb{N}$, $p\in\mathbb{N}^*$ thì $p^x\geq (p-1)x+1$ $(1)$ bằng quy nạp. Giả sử điều này đúng với $x=t$, tức là $p^t\geq (p-1)t+1$, khi đó $p^{t+1}=p.p^t\geq p(p-1)t+p=(p-1)(t+1)+1+t(p-1)^2\geq (p-1)(t+1)+1$, tức là điều này đúng với cả $x=t+1$, do đó $(1)$ được CM. Dấu $=$ xảy ra khi $x=1$

 

Quay trở lại bài toán, với $m-n\geq 1$, ta có $p^{m-n}\geq (p-1)(m-n)+1$. Kết hợp với $(\star)$ suy ra $p=2$ thỏa mãn kéo theo. Dấu $=$ xảy ra khi $m-n=1$. Hơn nữa ta cũng thu được $\frac{m}{n}=2$ nên $m=2,n=1$, hay $(a,b)=(4,2)$

Vậy $a=b$ hoặc $(a,b)=(4,2)$ và các hoán vị




#657909 GPT: $\sqrt[3]{x+6}+\sqrt{x-1}=x^{2...

Gửi bởi Chris yang trong 15-10-2016 - 14:30

2 câu bôi đỏ nhé.

Câu 9:

ĐKXĐ: $1\leq x\leq 3$. 

Ta có bài toán tương đương $2\sqrt{x-1}+2\sqrt{x-3}\leq 2x^3-8\sqrt{2x^3}+20\Leftrightarrow 2(\sqrt{x-1}+\sqrt{3-x})\leq (\sqrt{2x^3}-4)^2+4$

Thấy rằng $(\sqrt{2x^3}-4)^2+4\geq 4$ và theo BĐT Cauchy-Schwarz thì $2(\sqrt{x-1}+\sqrt{3-x})\leq 2\sqrt{(x-1+3-x)(1+1)}=4$. DO đó BPT luôn đúng với mọi $x$ nằm trong khoảng xác định, tức là $1\leq x\leq 3$

Câu 1: (chưa hoàn thành)

 

Đặt $x-1=a,y+1=b$.( $a,b>0$).Ta có $a^b=b^a$

$\Rightarrow b\log a=a\log b\Rightarrow \frac{\log a}{a}=\frac{\log b}{b}$. Đặt $b=a^r$ với $r\in\mathbb{R}$ thì $\frac{\log a}{a}=\frac{\log a^r}{a^r}=\frac{r\log a }{a^r}\Rightarrow r=a^{r-1}\Rightarrow a=r^{\frac{1}{r-1}}\Rightarrow b=r^{\frac{r}{r-1}}$

Đặt $\frac{1}{r-1}=t$, xét TH ta sẽ thu được bộ $(a,b)=(x-1,y+1)=\left ( 1+\frac{1}{t} \right )^{t},\left (1+\frac{1}{t} \right )^{t+1}$

Từ PT $(1)$ ta sẽ thu được $\sqrt{y+1}\geq 2\rightarrow y\geq 3$. Việc cần làm là ta đi chứng minh hàm $\left (1+\frac{1}{t} \right )^{t+1}$ là hàm nghịch biến trên hai khoảng, sẽ thu được $t\leq 1$. Lại có hàm $\left (1+\frac{1}{t} \right )^{t}$ là hàm đồng biến trên hai khoảng, kéo theo $x-1\leq 2\rightarrow  x\leq 3$ Mà $3,5\geq x\geq 3$ nên $x=3$, kéo theo $y=3$.

 

P.S: Cái này là ý tưởng của mình, chưa hoàn thành vì mình không biết nó đúng hay sai. Bạn nào thạo về phần đạo hàm, phương trình xem hộ mình xem ý tưởng của mình đúng hay không hộ cái

 

 

 




#649948 Nhận và khoe quà của Diễn đàn 2016

Gửi bởi Chris yang trong 16-08-2016 - 21:18

Em nhận được quà rồi ạ :D

 

Nói chung là VMEO IV đối với em là một trải nghiệm tuyệt vời! Dù đã bỏ toán ngót 1 năm , và trong cuộc thi này vẫn chưa dành tâm huyết hết mình cũng như sắp xếp được thời gian nhưng kết quả lần này cũng coi như không uổng phí 3 năm học toán hsg của em ^_^ Chưa bao giờ em nghĩ sẽ có một kỉ niệm nào đó gắn bó với VMF cho đến khi quyết định tham gia cuộc thi. Thực sự cảm ơn BTC, món quà rất tuyệt vời! :)

Hình gửi kèm

  • DDTH1.jpg



#647652 $(2y-1)\sqrt{1+x}+(2y+1)\sqrt{1-x}=2y$

Gửi bởi Chris yang trong 02-08-2016 - 17:07

Giải HPT trên tập số thực $\left\{\begin{matrix} \sqrt{9y^2+(2y+3)(y-x)}+4\sqrt{xy}=7x\\ (2y-1)\sqrt{1+x}+(2y+1)\sqrt{1-x}=2y\end{matrix}\right.$




#645760 Trung bình cộng của các ước của số $p^{m}q^{n...

Gửi bởi Chris yang trong 21-07-2016 - 00:53

Cho $p,q$ là 2 số nguyên tố phân biệt. Chứng minh rằng: tồn tại các số nguyên dương $m, n$ sao cho trung bình cộng của các ước của số $p^{m}q^{n}$ là $1$ số nguyên.

ĐKĐB tương đương với tồn tại $m,n$ để $A=\frac{(p^{m+1}-1)(q^{n+1}-1)}{(p-1)(q-1)(m+1)(n+1)}=\frac{(p^m+p^{m-1}+...+1)(q^n+q^{n-1}+...+1)}{(m+1)(n+1)}\in \mathbb{Z}$

+) Nếu $p=2$ và $q$ lẻ:  chọn $n=q$ thì $n+1|q^n+q^{n-1}+...+1$, ta chọn $m+1=\frac{q^{n}+q^{n-1}+...+1}{n+1}=1+q+q^2+...+q^{n-1}$. Khi đó $A\in\mathbb{Z}$. TH $q=2$ và $p$ lẻ tương tự như vậy.

+) Nếu cả $p,q$ đều lẻ, chọn $m=p,n=q$ hiển nhiên ta thu luôn được $A\in\mathbb{Z}$ :D

Do đó luôn tồn tại $m,n\in\mathbb{Z}^+$ thỏa mãn điều kiện trên.




#645751 Tìm n nguyên dương thoả mãn $3^{n-1}+5^{n-1}|3^n+5^n...

Gửi bởi Chris yang trong 20-07-2016 - 22:38

Bài 1:

 Tìm $n\in\mathbb{N^*}$ lẻ sao cho với $\left\{\begin{matrix} a,b|n \\ (a,b)=1 \end{matrix}\right.$ thì $a+b-1|n$

Bài 2:

 Tìm a,b,c nguyên dương thoả mãn $(a-1)(b-1)(c-1)|abc-1$

Bài 3:

 Tìm n nguyên dương thoả mãn $3^{n-1}+5^{n-1}|3^n+5^n$

 

Bài 1: Đặt $n=p^ta$ với $(p,a)=1$ và $p\in\mathbb{P}$, theo đkđb ta có ngay $p^t+a-1|p^ta\Rightarrow p^t+a-1|p^{2t}-p^t\rightarrow p^{2t}\geq 2p^t+a-1\Leftrightarrow p^{2t}\geq 2p^t+\frac{n}{p^t}-1>2\sqrt{2n}-1\Rightarrow p^{2t}\geq 2\sqrt{2n}$. Vì $p$ bất kỳ nên suy ra nếu $n$ có nhiều hơn hoặc bằng $4$ ước nguyên tố thì vô lý. Do đó ta xét các TH:

 

TH1: $n=p^t$ thì $a,b$ có một số là $1$. Giả sử đó là $b$ thì điều kiện trên luôn đúng

 

TH2: $n=p^t.q^m$ thì $p+q-1|p^t.q^m\Rightarrow p+q-1=p^{u}q^{v}$ với $u\leq t, v\leq m$ .Nếu $u,v\geq 1$ thì ta thu được $p=q=1$ (vô lý). Do đó một trong hai số $u,v$ bằng $0$. Giả sử $p>q$ Ta thấy $u=0$ vì nếu $v=0$ bắt buộc VT<VP. Do đó $p+q-1=q^v$. Lại có $m\geq v\geq 2$ nên $p+q^2-1=p^xq^y$. Bằng cách lập luận tương tự ở trên ta CM được rằng $x<2, x\neq 0\rightarrow x=1$  suy ra $q-1=q^{y-1}(p-q^{v-y})$, suy ra $y=1\Rightarrow p=q+1\Rightarrow p=3,q=2$, ta tìm được $n=12$

 

TH3: $n=p^tq^mr^s$. Giả sử $p>q>r$. Ta có $pq+r-1=p^{t'}q^{m'}r^{s'}$ ( $t'\leq t, m'\leq m, s'\leq s$ ) Lập luận tương tự TH2, thử lần lượt $a,b=0$,  ta thu được $pq+r-1=r^c$ $(1)$

Lại có $c\geq 2$ nên $pq+r^2-1=p^aq^br^c$ ( $a\leq t,b\leq m, c\leq s$), tương tự TH2 ta cũng có $pq+r^2-1=pqr^c$ $(2)$

Với $(1)$ và $(2)$ ta không tìm được bộ $(p,q,r)$ thỏa mãn, tức là không tồn tại $n$

 

Vậy $n=p^t$ hoặc $n=12$

 

Bài 2

Ta có $1\leq \frac{abc-1}{(a-1)(b-1)(c-1)}<\frac{abc}{(a-1)(b-1)(c-1)}\leq 2.2.2=6$

Không mất tính tổng quát giả sử $a\leq b\leq c$. Trước tiên dễ thấy TH $abc-1=(a-1)(b-1)(c-1)$ vô lý. 

Với các TH $abc-1=k(a-1)(b-1)(c-1)$ trong đó $k=2,3,4,5$ : ta giả sử $a\leq b\leq c$. Nếu $a>5$ kéo theo $b,c>5$, khi đó $\frac{abc-1}{(a-1)(b-1)(c-1)}\leq 1$ nên  $a=2,3,4$. Đến đây chỉ cần chịu khó xét các TH và thay giá trị của $a$ vào là có thể giải quyết dễ dàng.

 

Bài 3: 

ĐK tương đương $3^{n-1}+5^{n-1}|2.5^{n-1}\Rightarrow 3^{n-1}+5^{n-1}=2.5^t$ ( do $3^{n-1}+5^{n-1}$ chẵn) ( $t\leq n-1$)

Nếu $t\leq n-2$ thì hiển nhiên $\text{VT}>\text{VP}$ nên $t=n-1$, kéo theo $3^{n-1}=5^{n-1}$, suy ra $n=1$




#645465 Tìm $n$ để tổng các chữ số của $3n^2+n+1$ bằng 2016

Gửi bởi Chris yang trong 18-07-2016 - 22:56

Với mỗi số tự nhiên $n$, đặt $f(n)$ là tổng  các chữ số của $3n^2+n+1$

 

a. Tìm giá trị nhỏ nhất của $f(n)$

 

b. Tìm $n$ để $f(n)=2016$

Giải như sau:

a) 

Nếu $f(n)=1$ thì $3n^2+n+1=10^k$ với $k\geq 1$. Điều này hiển nhiên vô lý vì $3n^2+n+1$ luôn lẻ

Nếu $f(n)=2$ , ta xét 2 TH sau:

 +) TH1: $3n^2+n+1=2.10^k$: tương tự phía trên <vô lý>

 +) TH2:  $3n^2+n+1=10^a+10^b$ với $a>b\in\mathbb{N}$. Do $3n^2+n+1$ lẻ  nên $b=0$, hay $3n^2+n=n(3n+1)=10^a=2^a.5^a$. Phương trình tích với $(n,3n+1)=1$ ta dễ dàng thu được phương trình vô nghiệm

Nếu $f(n)=3$. Ta xét TH sau: $3n^2+n+1=2.10^k+1\Rightarrow n(3n+1)=2.10^k$. Dễ dàng giải PT trên ta thu được $n=8$ thỏa mãn, nghĩa là phương trình trên có nghiệm, hay tồn tại $n$ sao cho $f(n)=3$ là min

Vậy $f_{min}(n)=3$

b)  

Đoạn này cũng không biết cách làm nào khác ngoài mò :D

Dễ thấy $f(n)=2016=3n^2+n+1\pmod 9$ nên suy ra $n\equiv 5\pmod 9$

Khi đó thử đặt $n=10^t-5$. Ta có $3n^2+n+1=3.10^{2t}-29.10^t+71=2999...9710000...0071$ bao gồm một chữ số $2$, $t-2$ chữ số $9$, $2$ cặp $71$ và $t-2$ chữ số $0$

$\Rightarrow f(n)=2+9(t-2)+2(7+1)=2016$ kéo theo $t=224$

Vậy $n=10^{224}-5$ thì $f(n)=2016$




#643925 Tìm các số nguyên tố p,q

Gửi bởi Chris yang trong 07-07-2016 - 01:12

Tìm các số nguyên tố p.q sao cho: p^2-pq-q^3= 27

Phương trình $p(p-q)=(q+3)(q^2-3q+9)$

Dễ thấy $(p,q)=(7,2)$ là nghiệm. Nếu cả $p,q$ đều lẻ

TH1: $p|q+3\Rightarrow q+3\geq p\Rightarrow q^2-3q+9>q+3>p-q$, kéo theo $\text{VT}\leq \text{VP}$ ( vô lý)

TH2: $p|q^2-3q+9$. Đặt $q^2-3q+9=pk\rightarrow p-q=k(q+3)$. Ta thu được $q^2-q(k^2+k+3)+(9-3k^2)=0$. Với $k=1,2$ thì $q=3\Rightarrow p=9\not\in\mathbb{P}$.

Xét $k>2$, ta thấy $k|(2q-3)^2+27$, do đó nếu $r$ là ước nguyên tố lẻ nào đó của $k$ thì $\left ( \frac{-27}{r} \right )=\left ( \frac{-3}{r} \right )=1\Rightarrow r\equiv 1\pmod 6$, kéo theo $k\geq 7$. Nếu $k=7,8,9$ thì đều vô lý. Xét $k\geq 10$ 

Phương trình có nghiệm khi mà $T=(k^2+k+3)^2+4(3k^2-9)=k^4+2k^3+19k^2+6k-27$ là scp, với $k\geq 10$ dễ thấy $(k^2+k+8)^2<T<(k^2+k+9)^2$, do đó $T$ không thể là scp

Vậy $(p,q)=(7,2)$ 




#640396 Cho $n$ là một số nguyên dương lẻ. Chứng minh rằng $((n-1)^n+1...

Gửi bởi Chris yang trong 14-06-2016 - 23:53

Cho $n$ là một số nguyên dương lẻ. Chứng minh rằng $((n-1)^n+1)^2\mid n(n-1)^{(n-1)^n+1}+n$.

Bài này rất đơn giản: Vì $n$ lẻ nên $(n-1)^n+1$ lẻ. Gọi $p$ là ước nguyên tố bất kỳ của $(n-1)^n+1$. Ta cần có

$2v_p[(n-1)^n+1]\leq v_p(n)+v_p[(n-1)^{(n-1)^n+1}+1]$

$\Leftrightarrow 2v_p[(n-1)^n+1]\leq v_p(n)+v_p[(n-1)^n+1]+v_p[\frac{(n-1)^n+1}{n}]=2v_p[(n-1)^n+1]$

Điều này luôn đúng nên ta có đpcm




#640177 $\sum \frac{a^3+1}{b^2+c^2} \geq a+b+...

Gửi bởi Chris yang trong 14-06-2016 - 00:28

Mình chứng minh tương đương thì ra nhưng rất dài còn về phần cm theo bđt phụ chưa nghĩ ra mọi người đưa ra ý kiến đi !

 

:ukliam2:  :ukliam2:  :ukliam2:  :ukliam2:  :ukliam2:  :ukliam2:  :ukliam2:  :ukliam2:  :ukliam2:  :ukliam2:

Thỉnh cách CM tương đương của bạn! Mình sử dụng cũng có quá trình biến đổi tương đương nhưng không dài cho lắm =))

 

Ta đưa về BĐT đồng bậc, tức là với $a,b,c>0$, cần chứng minh

 

$\frac{a^3+abc}{b^2+c^2}+\frac{b^3+abc}{c^2+a^2}+\frac{c^3+abc}{a^2+b^2}\geq a+b+c$

 

$\Leftrightarrow \left ( \frac{a^3+abc}{b^2+c^2}-a \right )+\left ( \frac{b^3+abc}{c^2+a^2}-b \right )+\left ( \frac{c^3+abc}{a^2+b^2}-c \right )\geq 0$

 

$\Leftrightarrow \frac{a(a-b)(a-c)}{b^2+c^2}+\frac{b(b-c)(b-a)}{c^2+a^2}+\frac{c(c-a)(c-b)}{a^2+b^2}\geq 0$ $(\star)$

 

Giờ thì giả sử $a\geq b\geq c$ thì $\frac{a}{b^2+c^2}\geq \frac{b}{c^2+a^2}$

 

Khi đó, BĐT $(\star)$ hiển nhiên đúng theo BĐT Vornicu- Schur

 

Dấu $=$ xảy ra khi $a=b=c=1$




#640065 Đề thi tuyển sinh vào lớp 10 THPT chuyên Vĩnh Phúc năm học 2016-2017 (vòng 2)

Gửi bởi Chris yang trong 13-06-2016 - 16:01

Bài 2:

a) Phương trình tương đương $(\sqrt{4x-3}-2x)^2=x^2\Leftrightarrow (\sqrt{4x-3}-x)(\sqrt{4x-3}-3x)=0$ 

Dễ dàng giải PT trên ta thu được $x=1,3$ là nghiệm

b)(Tính chia hết áp dụng cho cả số âm và dương)

Dễ thấy $y|x$ nên đặt $x=ty$. Ta có $t^2=ty+y^4+2y^2$. Gọi $m=\gcd(t,y)$ nên $t=t_1m,y=y_1m$ với $(t_1,y_1)=1$ $(1)$

Khi đó $t_1^2=y_1(t_1+y_1^3m^2+2y_1)$, kết hợp $(1)$ suy ra $y_1=\pm 1$

TH1 $y_1=1$ thì $t_1^2=t_1+m^2+2\rightarrow (2t_1-2m-1)(2t_1+2m-1)=9$

TH2: $y_1=-1$ thì $(2t_1-2m+1)(2t_1+2m+1)=9$

Hai phương trình tích trên đều có thể giải nghiệm nguyên dễ dàng!

 

Bài 4:

a) 

Do tứ giác $BMDF$ và $MECD$ nội tiếp nên $\angle BFD=\angle BMA=\angle DMC=\angle DEC$ $(1)$

Và tứ giác $ABCD$ nội tiếp nên $\angle FBD=\angle DCE$ $(2)$

Từ $(1), (2)\Rightarrow \bigtriangleup BDF\sim \bigtriangleup CDE$ 

$\Rightarrow \angle FDB=\angle EDC$. Mà $\angle FDB=\angle FMB$ và $\angle EDC=\angle EMC$ nên $\angle FMB=\angle EMC$, kéo theo $\overline{F,M,E}$

b) Từ $A$ kẻ tiếp tuyến $Ax$ của $(O)$ ( $x$ nằm cùng phía $B$ so với $OA$)

Ta có $\angle xAF=\angle ACB=\angle BDA=\angle AFM=\angle AFE$ ( do $\overline{F,M,E}$ và $BMDF$ nội tiếp)

$\Rightarrow Ax\parallel EF$. Hiển nhiên $Ax\perp OA\rightarrow OA\perp EF$

c) 

Để $PQ\parallel BC$ cần có $\frac{BQ}{QN}=\frac{PC}{PN}$

Theo tính chất đường phân giác $\frac{BQ}{QN}=\frac{BF}{FN},\frac{PC}{PN}=\frac{EC}{EN}$, nên cần chứng minh $ \frac{BF}{FN}=\frac{EC}{EN}\Leftrightarrow \frac{BF}{CE}=\frac{FN}{EN}=\frac{AF}{AE}=\frac{AC}{AB}$

$\Leftrightarrow BF.AB=AE.AC\Leftrightarrow AB.AF+AC.AE=AB^2+AC^2$

Thấy rằng $AB.AF=AE.AC=AM.AD=AM^2+AM.MD=AM^2+MB.MC=AM^2+\frac{BC^2}{4}$ nên ta đi chứng minh $AB^2+AC^2=2AM^2+\frac{BC^2}{2}$

Đến đây kẻ đường cao $AH$ của tam giác $ABC$ rồi sử dụng định lý Pitago ta có đpcm

 

 

 

 

 




#639370 $(ab+c^2)(bc+a^2)(ca+b^2) \ge abc(a+b)(b+c)(a+c)$

Gửi bởi Chris yang trong 10-06-2016 - 16:31

Cho $a,b,c>0$ chứng tỏ : $(ab+c^2)(bc+a^2)(ca+b^2) \ge abc(a+b)(b+c)(a+c)$ 

Áp dụng BĐT Cauchy Schwarz

$(ab+c^2)\left ( \frac{a}{b}+1 \right )\geq (a+c)^2$

Tương tự cũng có $(bc+a^2)\left ( \frac{b}{c}+1 \right )\geq (b+a)^2$ và $(ca+b^2)\left ( \frac{c}{a}+1 \right )\geq (b+c)^2$

Nhân theo vế ta có đpcm




#639052 $3^{x}- 2^{y}= 19^{z}$

Gửi bởi Chris yang trong 09-06-2016 - 00:26

Tìm tất cả các bộ số nguyên dương (x, y, z) thoả mãn

$3^{x}- 2^{y}= 19^{z}$

Thấy rằng $x\geq 2$ nên $19^z+2^y\equiv 1+2^y\equiv 0\pmod 9$ kéo theo $y\equiv 3\pmod 6$ suy ra $y\geq 3$

Khi đó $3^x-19^z\equiv (-1)^x-(-1)^z\equiv 0\pmod 4$ nên $x,z$ cùng tính chẵn lẻ

TH1: $x,z$ cùng chẵn. Đặt $x=2m,z=2n$ thì $(3^m-19^n)(3^m+19^n)=2^y$. Dễ dàng giải PT tích trên ta không thu được nghiệm nào thỏa mãn

TH2: $x,z$ cùng lẻ

Ta có $19^z+2^y=3^x\equiv 0\pmod {27}$. Xét modulo $6$ cho $z$ ta có $z$ có dạng $6k+1,6k+5$ ( trường hợp $z=6k+3$ kéo theo $7|3^x$ - vô lý). Xét modulo $18$ cho $y$ do $y\equiv 3\pmod 6$ nên $y$ có dạng $18l+3,18l+9,18l+15$. Thử ta thu được bộ $(z,y)=(6k+1,18l+3)$ và $(z,y)=(6k+5,18l+15)$

+) Nếu $(z,y)=(6k+5,18l+15)$ thì $3^x\equiv 4\pmod 7$. Xét modulo $6$ cho $x$ ta thu đươc $x\equiv 4\pmod 6$ ( vô lý vì $x$ chẵn)

+) Nếu $(z,y)=(6k+1,18l+3)$ thì $3^x\equiv -1\pmod 7$ kéo theo $3|x$

Như vậy, $3|x,y$ nên đặt $x=3t,y=3k$. Ta có  $(3^t-2^k)(3^{2t}+3^t2^k+2^{2k})=19^z$. 

Bằng cách đặt $3^t-2^k=19^u, 3^{2t}+3^t2^k+2^{2k}=19^v$ $(1)$ ta thu được phương trình $3^t-2^k=1$. Đây là một phương trình quen thuộc có nghiệm $(t,k)=(1,1),(2,3)$. Thử lại với $(1)$ ta được $(t,k)=(1,1)$ tức $(x,y)=(3,3)$

Vậy bộ $(x,y,z)=(3,3,1)$